\documentclass[11pt]{article} \usepackage{personal_commands} \usepackage[italian]{babel} \title{\textbf{Note del corso di Analisi Matematica 1}} \author{Gabriel Antonio Videtta} \date{31 marzo, 4, 18 e 20 aprile 2023} \begin{document} \maketitle \begin{center} \Large \textbf{Teoria sulle derivate} \end{center} \begin{definition} (derivata) Sia $f : X \subseteq \RR \to \RR$. Si definisce allora \textbf{derivata} di $f$ in $\xbar \in X$ punto di accumulazione, se esiste, il seguente limite: \[Df(\xbar) = f'(\xbar) = \lim_{h \to 0} \frac{f(\xbar + h) - f(\xbar)}{h} = \lim_{x \to \xbar} \frac{f(x) - f(\xbar)}{x - \xbar}.\] \vskip 0.05in Qualora tale limite non esista, si dirà che non esiste la derivata di $f$ in $\xbar$. Si definisce anche $f' : D \subseteq X \to \RRbar$ come la funzione derivata, la quale associa ogni punto $\xbar$ in cui la derivata di $f$ esiste al valore del limite computato in $\xbar$. \end{definition} \begin{definition} $\xbar \in X$ si dice \textbf{derivabile} se e solo se esiste la derivata di $f$ in $\xbar$ e $f'(\xbar)$ è finito. \end{definition} \begin{remark}\nl \li L'insieme $D$ può essere vuoto. \\ \li Si definisce $f^{(n)}(\xbar)$ come la derivata $n$-esima di $f$ in $\xbar$. \\ \li Si definisce per convenzione $f^{(0)}(x) = f(x)$. \\ \li L'operazione di derivata è un operatore lineare. \\ \end{remark} \begin{definition} (derivata destra e sinistra) Dato $\xbar$ punto di accumulazione destro di $X$, si definisce allora \textbf{derivata destra} di $f$ in $\xbar \in X$, se esiste, il seguente limite: \[D_+ f(\xbar) = f_+'(\xbar) = \lim_{h \to 0^+} \frac{f(\xbar + h) - f(\xbar)}{h} = \lim_{x \to \xbar^+} \frac{f(x) - f(\xbar)}{x - \xbar}.\] \vskip 0.05in Qualora tale limite non esista, si dirà che non esiste la derivata destra di $f$ in $\xbar$. Analogamente, per un punto di accumulazione sinistro $\xbar \in X$, si definisce la \textbf{derivata sinistra} di $f$ in $\xbar \in X$, se esiste, il seguente limite: \[D_- f(\xbar) = f_-'(\xbar) = \lim_{h \to 0^-} \frac{f(\xbar + h) - f(\xbar)}{h} = \lim_{x \to \xbar^-} \frac{f(x) - f(\xbar)}{x - \xbar}.\] \end{definition} \begin{remark}\nl \li Se esistono sia la derivata sinistra che destra di $f$ in $\xbar$ e coincidono, allora la derivata di $f$ in $\xbar$ esiste e coincide con il valore di entrambe le due derivate. \\ \li Vale anche il viceversa, se $\xbar$ è un punto di accumulazione sia destro che sinistro: se esiste la derivata di $f$ in $\xbar$, allora sia la derivata sinistra che destra esistono e coincidono con la derivata. \end{remark} \begin{definition} Si dice che $f : X \to \RR$ è derivabile se è derivabile $\forall x \in X$. \end{definition} \begin{definition} Si dice che $f \in \cc^1$ se è derivabile e la sua funzione derivata è continua. In generale, si dice che $f \in \cc^n$ se è derivabile $n$ volte e ogni sua derivata, fino alla $n$-esima, è continua. Si pone $f \in \cc^\infty$ se $f$ è derivabile per un numero arbitrario di volte e ogni sua derivata è continua. \end{definition} \begin{proposition} Sia $f : X \to \RR$ e sia $\xbar \in X$ un punto di accumulazione di $X$. Allora: \begin{enumerate}[(i)] \item $f$ derivabile in $\xbar$ $\implies$ $f(\xbar + h) = f(\xbar) + f'(\xbar) h + o(h)$. \item Se esiste $a$ tale che $f(\xbar + h) = f(\xbar) + ah + o(h)$, allora $f$ è derivabile in $\xbar$ e $f'(\xbar) = a$. \end{enumerate} \end{proposition} \begin{proof} Se $f$ è derivabile in $\xbar$, allora $\lim_{h \to 0} \frac{f(\xbar + h) - f(\xbar) - f'(\xbar) h}{h} = \lim_{h \to 0} \frac{f(\xbar + h) - f(\xbar)}{h} - f'(\xbar) = f'(\xbar) - f'(\xbar) = 0$, da cui la prima tesi. \\ Inoltre, se esiste $a$ come nelle ipotesi, $\lim_{h \to 0} \frac{f(\xbar + h) - f(\xbar)}{h} =\lim_{h \to 0} \frac{ah + o(h)}{h} = a + \lim_{h \to 0} \frac{o(h)}{h} = a + 0 = a$, quindi $f$ è derivabile in $\xbar$ e $f'(\xbar) = a$. \end{proof} \begin{corollary} Se $f$ è derivabile in $\xbar$, allora $f$ è anche continua in $\xbar$. \end{corollary} \begin{proof} Infatti, poiché $f(x) = f(\xbar) + f'(\xbar) (x - \xbar) + o(x-\xbar)$, $\lim_{x \to \xbar} f(x) = \lim_{x \to \xbar} f(\xbar) + \lim_{x \to \xbar} f'(\xbar)(x-\xbar) + \lim_{x \to \xbar} o(x - \xbar) = \lim_{x \to \xbar} f(\xbar) = f(\xbar)$, e quindi $f$ è continua in $\xbar$. \end{proof} %TODO: trovare esempio di derivabilità infinita e non continuità \begin{proposition} Siano $f_1$, $f_2 : X \to \RR$ entrambe derivabili in $\xbar$. Allora: \begin{enumerate}[(i)] \item $(f_1 + f_2)'(\xbar) = f_1'(\xbar) + f_2'(\xbar)$, \item $(f_1f_2)'(\xbar)= f_1(\xbar) f_2'(\xbar) + f_1'(\xbar) f_2(\xbar)$. \end{enumerate} \end{proposition} \begin{proof}Poiché $f_1$ ed $f_2$ sono derivabili in $\xbar$, vale che: \[ f_1(\xbar + h) = f_1(\xbar) + f_1'(\xbar) h + o(h), \qquad f_2(\xbar + h) = f_2(\xbar) + f_2'(\xbar) h + o(h). \] \begin{enumerate}[(i)] \item $(f_1 + f_2)(\xbar + h) = (f_1 + f_2)(\xbar) + (f_1' + f_2')(\xbar) h + o(h)$. Quindi, per la proposizione precedente, $(f_1 + f_2)'(\xbar) = (f_1' + f_2')(\xbar) = f_1'(\xbar) + f_2'(\xbar)$. \item $(f_1 f_2)(\xbar + h) = (f_1 f_2)(\xbar) + (f_1(\xbar)f_2'(\xbar) + f_1'(\xbar) f_2(\xbar)) h + \underbrace{(f_1(\xbar) + f_2(\xbar)) o(h) + (f_1'f_2')(\xbar) h^2 + (f_1'(\xbar) + f_2'(\xbar))h \cdot o(h) + o^2(h))}_{=o(h)} = (f_1 f_2)(\xbar) + (f_1(\xbar)f_2'(\xbar) + f_1'(\xbar) f_2(\xbar)) h + o(h)$. Quindi, per la proposizione precedente, $(f_1 f_2)'(\xbar) = f_1(\xbar)f_2'(\xbar) + f_1'(\xbar) f_2(\xbar)$. \end{enumerate} \end{proof} \begin{proposition} Siano $f : X \to Y$ e $g : Y \to \RR$, con $f$ derivabile in $\xbar$ e $g$ derivabile in $\ybar := f(\xbar)$. Allora $g \circ f$ è derivabile in $\xbar$ e $(g \circ f)'(\xbar) = f'(\xbar) g'(\ybar)$. \end{proposition} \begin{proof} Poiché $f'(\xbar)$ è finito, $f(\xbar + h) = \ybar + f'(\xbar) h + o(h)$. Analogamente, $g(\ybar + h) = g(\ybar) + g'(\ybar) h + o(h)$. Allora $g(f(\xbar + h)) = g(\ybar + (f'(\xbar) h + o(h))) = g(\ybar) + g'(\ybar) (f'(\xbar) h + o(h)) + o(f'(\xbar) h + o(h)) = g(\ybar) + g'(\ybar) f'(\xbar) h + o(h) + o(f'(\xbar) h + o(h))$. \\ Si osserva che $\lim_{h \to 0} \frac{o(f'(\xbar) h + o(h))}{h} = \lim_{h \to 0} \frac{o(f'(\xbar) h + o(h))}{f'(\xbar) h + o(h)} \frac{f'(\xbar) h + o(h)}{h} = \lim_{h \to 0} \frac{o(f'(\xbar) h + o(h))}{f'(\xbar) h + o(h)} \lim_{h \to 0} \frac{f'(\xbar) h + o(h)}{h} = 0 \cdot f'(\xbar) = 0$, e quindi che $o(f'(\xbar) h + o(h)) = o(h)$. Allora $g(f(\xbar + h)) = g(\ybar) + g'(\ybar) f'(\xbar) h + o(h)$, da cui si conclude che $(g \circ f)'(\xbar) = g'(\ybar) f'(\xbar)$. \end{proof} \begin{proposition} Sia $f : X \to Y$ con inversa $g : Y \to X$. Sia $f$ derivabile in $\xbar$ con $f'(\xbar) \neq 0$. Sia $g$ continua in $\ybar = f(\xbar)$. Allora: \begin{enumerate}[(i)] \item $\ybar$ è un punto di accumulazione di $Y$, \item $g$ è derivabile in $\ybar$ e $g'(\ybar) = \frac{1}{f'(\xbar)}$. \end{enumerate} \end{proposition} \begin{proof} Si dimostrano i due risultati separatamente. \begin{enumerate}[(i)] \item Poichè $f$ è derivabile in $\xbar$, $f$ è continua in $\xbar$. Quindi per ogni intorno $I$ di $\ybar$, esiste un intorno $J$ di $\xbar$ tale per cui $f(I \cap X \setminus \{ \xbar \}) \subseteq J$. Inoltre, $I \cap X \setminus \{\xbar\}$ non è mai vuoto, dacché, essendo $f$ derivabile in $\xbar$, $\xbar$ è un punto di accumulazione di $X$. Quindi $J$ contiene in particolare un immagine di $f$ in esso, e quindi un punto di $Y$; inoltre, tale punto è diverso da $\ybar$ dal momento che $f$ è iniettiva, essendo bigettiva. Quindi $\ybar$ è un punto di accumulazione. \item Poiché $f$ è derivabile in $g(\ybar)$, $\ybar + h = f(g(\ybar + h)) = f(g(\ybar) + (\underbrace{g(\ybar + h) - g(\ybar)}_k)) = \ybar + f'(\xbar) k + o(k)$, ossia vale che: \[ h = f'(\xbar) k + o(k). \] Dal momento che $g$ è continua in $\ybar$, $k \tends{h \to 0} 0$, e quindi $o(k) \tends{h \to 0} 0$. Quindi, per $h \to 0$, $k \sim \frac{h}{f'(\xbar)}$. Si conclude dunque che $\lim_{h \to 0} \frac{g(\ybar + h) - g(\ybar)}{h} = \lim_{h \to 0} \frac{k}{h} = \frac{1}{f'(\xbar)}$. \end{enumerate} \end{proof} \begin{example} La continuità è necessaria nelle scorse ipotesi. Si può costruire infatti una funzione del tipo: \[ f(x) = \system{x & \se x \geq 0, \\ -(x+2) & \se -2 < x \leq -1.} \] \vskip 0.05in dove $f'(0) = 1$, $f$ è invertibile, ma la derivata di $g$ in $0$ non esiste ($D_+ g(0) = 1$, ma $D_- g(0) = +\infty$). \end{example} \begin{theorem} (di Fermat) Sia $I$ intervallo, $f : I \to \RR$, $\xbar$ interno a $I$ punto di massimo o minimo locale con $f$ derivabile in $\xbar$, allora $f'(\xbar) = 0$. \end{theorem} \begin{proof} Poiché $I$ è un intervallo e $\xbar$ è interno a $I$, $\xbar$ è sia punto di accumulazione sinistro che punto di accumulazione destro di $I$. Dal momento che $f$ è derivabile in $\xbar$, esistono sia la derivata destra che la derivata sinistra in $\xbar$. \\ Si assuma che $\xbar$ è un punto di massimo locale (altrimenti è sufficiente considerare $g = -f$). Allora esiste un intorno $J$ di $\xbar$ tale per cui $x \in J \implies f(x) - f(\xbar) \leq 0$. Sia dunque $J_+$ l'intorno destro relativo a $J$, e sia $J_-$ quello sinistro. \\ Poiché $\xbar = \inf J_+$, esiste una successione $\{x_n\} \subseteq J_+ \setminus \{\xbar\}$ tale per cui $x_n \tendston \xbar$. Dal momento che allora $f$ è derivabile in $\xbar$, $f$ è anche continua in $\xbar$, e quindi si ricava che $f(x_n) \tendston f(\xbar)$. Si osserva dunque che $f(x_n) - f(\xbar) \leq 0$ e $x_n - \xbar > 0 \implies \frac{f(x_n) - f(\xbar)}{x_n - \xbar} \leq 0$, da cui, per il teorema della permanenza del segno, si ricava che $L_+ = \lim_{n \to \infty} \frac{f(x_n) - f(\xbar)}{x_n - \xbar} \leq 0$. \\ Allora, dal momento che $f$ è derivabile in $\xbar$ e che la derivata destra deve coincidere con la derivata classica, $f'(\xbar) = \lim_{x \to \xbar^+} \frac{f(x) - f(\xbar)}{x-\xbar} = \lim_{n \to \infty} \frac{f(x_n) - f(\xbar)}{x-\xbar} = L_+ \leq 0$. \\ Analogamente si ricava che $f'(\xbar) \geq 0$, e quindi che $f'(\xbar)$ è necessariamente pari a zero, da cui la tesi. \end{proof} \begin{remark} \nl \li Si può facilmente generalizzare il teorema di Fermat assumendo ipotesi più deboli. Sia infatti $x_M$ un punto di massimo locale e sia $f$ continua in $x_M$, allora, qualora esistano, $D_+ f(x_M) \leq 0$ e $D_- f(x_M) \geq 0$. Analogamente si estende la proposizione a $x_m$ punto di minimo locale. \end{remark} \begin{theorem} (di Rolle) Sia $I = [a, b] \subset \RR$ e sia $f : I \to \RR$ tale che $f$ sia continua su $I$, che $f(a) = f(b)$ e che $f$ sia derivabile in $[a, b]$. Allora $\exists \xbar \in (a, b)$ tale che $f'(\xbar) = 0$. \end{theorem} \begin{proof} Per il teorema di Weierstrass $f$ ammette un punto di massimo $M$ e uno di minimo $m$ in $I$. Se $f(a) = M$ e $f(b) = m$ o viceversa, la funzione $f$ è costante in $I$, e quindi per ogni punto in $(a, b)$ la derivata è nulla. Altrimenti, sicuramente uno tra il punto di massimo e quello di minimo appartiene a $(a, b)$. Sia $\xbar$ tale punto. Allora, per il teorema di Fermat, $f'(\xbar) = 0$, da cui la tesi. \end{proof} \begin{theorem} (di Cauchy) Sia $I = [a, b] \subset \RR$ e siano $f$, $g: I \to \RR$ continue su $I$ e derivabili in $(a, b)$, con $g'$ non nulla in $(a, b)$ e $g(a) \neq g(b)$. Allora $\exists \xbar \in (a, b)$ tale che $\frac{f'(\xbar)}{g'(\xbar)} = \frac{f(b) - f(a)}{g(b)-g(a)}$. \end{theorem} \begin{proof} Si consideri la funzione $h : I \to \RR$ tale che $h(x) = f(x) - \left(\frac{f(b) - f(a)}{g(b) - g(a)} (g(x) - g(a)) + f(a)\right)$. Si osserva che $h$, essendo una somma di funzioni continue su $I$ e derivabili in $(a, b)$, è anch'essa continua su $I$ e derivabile in $(a, b)$. Inoltre $h(a) = h(b) = 0$. Quindi, per il teorema di Rolle, $\exists \xbar \in (a, b) \mid h'(\xbar) = 0 \implies \frac{f'(\xbar)}{g'(\xbar)} = \frac{f(b)-f(a)}{g(b)-g(a)}$, da cui la tesi. \end{proof} \begin{theorem} (di Lagrange) Sia $I = [a, b] \subset \RR$ e sia $f: I \to \RR$ tale che $f$ sia continua su $I$ e che $f$ sia derivabile in $(a, b)$. Allora $\exists \xbar \in (a, b)$ tale che $f'(\xbar) = \frac{f(b) - f(a)}{b-a}$, ossia tale per cui la retta tangente a $f$ in $\xbar$ è parallela alla secante che passa per $(a, f(a))$ e $(b, f(b))$. \end{theorem} \begin{proof} Si consideri $g(x) = x$. $g$ è continua in $[a, b]$ e derivabile in $(a, b)$, con derivata sempre non nulla in tale intervallo. Allora, per il teorema di Cauchy, $\exists \xbar \in (a, b) \mid f'(\xbar) = \frac{f(b)-f(a)}{b-a}$, da cui la tesi. \end{proof} \begin{proposition} Sia $I = [a, b] \subset \RR$ e sia $f : I \to \RR$ tale che $f$ sia continua su $I$ e che $f$ sia derivabile in $(a, b)$, con derivata non negativa. Allora $f$ è crescente in $[a, b]$. Analogamente, se la derivata è non positiva, $f$ è decrescente. \end{proposition} \begin{proof} Senza perdita di generalità si dimostra il caso in cui la derivata di $f$ in $(a, b)$ è non negativa (altrimenti è sufficiente considerare $g = -f$). Si considerino $c < d \in I$. Allora, per il teorema di Lagrange, $\exists \xbar \in (c, d) \mid f'(c) = \frac{f(d) - f(c)}{d-c} \implies f(d) - f(c) = \underbrace{f'(c) (d-c)}_{\geq 0} \implies f(d) \geq f(c)$, e quindi $f$ è crescente in $I$, da cui la tesi. \end{proof} \begin{proposition} Sia $I \subset \RR$ un intervallo e sia $f : I \to \RR$ tale che $f$ sia derivabile in $I$. Allora $f$ è convessa se e solo se la derivata è crescente. \end{proposition} \begin{proof} Si dimostrano le due implicazioni separatamente. \\ \rightproof Siano $x_0$, $x_1 \in I$ con $x_0 < x_1$. Sia $h$ positivo tale che $x_0 < x_0 + h \leq x_1$. Allora $x_0 + h = (1- \lambda) x_0 + \lambda x_1$ con $\lambda = \frac{h}{x_1 - x_0}$. Allora, poiché $f$ è convessa, $f(x_0 + h) \leq (1- \lambda) f(x_0) + f(x_1) \leq f(x_0) + \frac{h}{x_1 - x_0} \left( f(x_1) - f(x_0) \right)$, da cui si ricava che: \[ \frac{f(x_0 + h) - f(x_0)}{h} \leq \frac{f(x_1) - f(x_0)}{x_1 - x_0}. \] Quindi, passando al limite, $f'(x_0) \leq \frac{f(x_1) - f(x_0)}{x_1 - x_0}$. Analogamente si dimostra che $\frac{f(x_1) - f(x_0)}{x_1 - x_0} \leq f'(x_1)$. Si conclude dunque che $f'(x_1) \geq f'(x_0)$, ossia che $f'$ è crescente. \\ \leftproof Siano $x_0$, $x_1 \in I$ con $x_0 < x_1$. Si considera $x = (1 - \lambda) x_0 + \lambda x_1 \in (x_0, x_1)$ con $0 < \lambda < 1$. Per il teorema di Lagrange $\exists \tilde{x_0} \in (x_0, x)$ tale che $f'(\tilde{x_0}) = \frac{f(x) - f(x_0)}{x - x_0}$. Analogamente $\exists \tilde{x_1} \in (x, x_1)$ tale che $f'(\tilde{x_1}) = \frac{f(x_1) - f(x)}{x_1 - x}$. Poiché allora per ipotesi la derivata $f'$ è crescente, si ricava che: \[ \frac{f(x_1) - f(x)}{x_1 - x} \geq \frac{f(x) - f(x_0)}{x - x_0}, \] da cui si conclude che: \[ f(x) \leq (1-\lambda) f(x_0) + \lambda f(x_1), \] \vskip 0.05in ossia che vale la disuguaglianza di Jensen, e quindi che $f$ è convessa, da cui la tesi. \end{proof} \begin{remark}\nl \li L'interpretazione geometrica del teorema di Cauchy, rispetto a quella di Lagrange, è leggermente più complicata. Si consideri la curva continua $\gamma : \RR \to \RR^2$ tale che $\gamma(t) =(g(t), f(t))$. Si osserva che il coefficiente della retta tangente in $\xbar$ per $\gamma$ è dato da $\lim_{h \to 0} \frac{f(\xbar + h) - f(\xbar)}{g(\xbar + h) - g(\xbar)}$, che, sotto le ipotesi del teorema di Cauchy, può essere riscritto come $\frac{f'(\xbar)}{g'(\xbar)}$. Allora, il teorema di Cauchy asserisce che esiste un punto della curva $\gamma$ tale per cui la retta tangente alla curva in quel punto è parallela alla secante passante per $(g(a), f(a))$ e $(g(b), f(b))$. \\ \li Inoltre $f$ è strettamente crescente in $I$ se $f' \geq 0$ e non esistono intervalli di punti stazionari. Analogamente se $f' < 0$ in $I$ e non esistono ancora tali intervalli, $f$ è strettamente decrescente in $I$. \end{remark} \begin{exercise} Si descriva un insieme $X$ tale che i suoi unici punti di accumulazione siano $\pm 1$. \end{exercise} \begin{solution} Si consideri $X = \{1 + \frac{1}{n}\} \cup \{-1 - \frac{1}{n}\}$, al variare di $n \in \NN$. Sia $J = [1 - \eps, 1 + \eps]$ un intorno di $1$. Allora $1 + \frac{1}{n} \in J$ per $n > \frac{1}{\eps}$, da cui si ricava che $1$ è un punto di accumulazione di $X$; analogamente si verifica che $-1$ è un punto di accumulazione di $X$. Si consideri adesso l'intorno $J = \left[1 + \frac{1}{n} - \frac{1}{2} \left( \frac{1}{n} - \frac{1}{n+1} \right), 1 + \frac{1}{n} + \frac{1}{2} \left( \frac{1}{n} - \frac{1}{n+1} \right) \right]$. Si verifica che nessun punto di $X$, oltre $1 + \frac{1}{n}$ appartiene a $J$, e quindi $1 + \frac{1}{n}$ non è punto di accumulazione di $X$. Analogamente non lo è alcun numero della forma $-1 - \frac{1}{n}$. \end{solution} \begin{exercise} Sia $f : X \to \RRbar$ continua in $\xbar$ e sia $a < f(\xbar)$. Allora esiste $J$ intorno di $\xbar$ tale che $a < f(x)$ $\forall x \in J \cap X \setminus \{\xbar\}$. \end{exercise} \begin{solution} Si consideri $g : X \to \RRbar$ tale che $g(x) = f(x) - a$. Poiché $g$ è una somma di funzioni continue in $\xbar$, anch'essa è continua in $g$. Allora, poiché $g(\xbar) > 0$, per il teorema della permanenza del segno, esiste un intorno $J$ di $\xbar$ tale per cui $g(x) > 0$ $\forall x \in J$, ossia tale per cui $f(x) > a$ $\forall x \in J$, da cui la tesi. \end{solution} \begin{exercise} Sia $X \subseteq \RRbar$ e sia $\xbar$ punto di accumulazione di $X$. Siano $f_1$, $f_2 : X \to \RRbar$. Si dimostri allora che: \begin{enumerate}[(i)] \item se $f_1 \tendsto{\xbar} +\infty$ e $f_2$ è limitata inferiormente in un intorno $J$ di $\xbar$, allora $f_1(x) + f_2(x) \tendsto{\xbar} +\infty$; \item se $f_1 \tendsto{\xbar} 0$ e $f_2$ è limitata in un intorno $J$ di $\xbar$, allora $f_1 f_2(x) \tendsto{\xbar} 0$; \item se $f_1 \tendsto{\xbar} +\infty$ ed $f_2$ è limitata inferiormente da una costante positiva $c$ in un intorno $J$ di $\xbar$, allora $f_1 f_2 \tendsto{\xbar} +\infty$. \end{enumerate} \end{exercise} \begin{solution} Si dimostrano i tre risultati separatamente. \begin{enumerate}[(i)] \item Sia $c$ la costante tale per cui $f_2(x) \geq c$ $\forall x \in J \cap X$. Sia $I = [a, \infty]$ un intorno di $+\infty$. Se $c < 0$, poiché $f_1 \tendsto{\xbar} +\infty$, esiste un intorno $J'$ tale per cui $f_1(J' \cap X \setminus \{ \xbar \}) \subseteq [a-c, \infty] \subseteq I$. Sia dunque $Z = J \cap J'$. Allora $(f_1 + f_2)(x) = f_1(x) + f_2(x) \geq a-c+c = a$ $\forall x \in Z$, da cui si conclude che $(f_1 + f_2)(Z \cap X \setminus \{\xbar\}) \subseteq I$. Se invece $c \geq 0$, è sufficiente considerare un intorno $J'$ di $\xbar$ tale per cui $f_1(J' \cap X \setminus \{\xbar\}) \subseteq I$, da cui $(f_1 + f_2)(x) = f_1(x) + f_2(x) \geq a+c \geq a$ $\forall x \in Z \implies (f_1+f_2)(Z \cap X \setminus \{\xbar\}) \subseteq I$, da cui la tesi. \item Poiché $f_2$ è limitata in $J$, esistono delle costanti finite $a$, $b \in \RR$ tali per cui $a \leq f_2(x) \leq b$ $\forall x \in J$. Sia $I = [-\eps, \eps]$ un intorno di $0$, con $\eps > 0$. Si consideri $c := \max\{\abs a, \abs b\}$. Allora vale che $-c \leq f_2(x) \leq c$ $\forall x \in J$. Poiché $f_1 \tendsto{\xbar} 0$, esiste un intorno $J'$ di $\xbar$ tale per cui $f(J' \cap X \setminus \{\xbar\}) \subseteq \left[-\frac{\eps}{c}, \frac{\eps}{c}\right]$. Si consideri ora $Z := J \cap J'$: vale allora che $\abs{(f_1 f_2)(x)} = \abs{f_1(x) f_2(x)} \leq c \frac{\eps}{c} = \eps$ $\forall x \in Z \cap X \setminus \{\xbar\}$. Si conclude dunque che $(f_1f_2)(Z \cap X \setminus \{\xbar\}) \subseteq I$, da cui la tesi. \item Sia $I = [a, \infty]$ un intorno di $+\infty$. Allora, poiché $f_1 \tendsto{\xbar} +\infty$, esiste un intorno $J'$ di $\xbar$ tale per cui $f_1(J' \cap X \setminus \{\xbar\}) \subseteq [\abs a, \infty] \subseteq I$. Si consideri dunque $Z := J \cap J'$: vale dunque che $(f_1f_2)(x) = f_1(x) f_2(x) \geq \abs{a}c \geq a$ $\forall x \in Z \cap X \setminus \{\xbar\}$. Si conclude allora che $(f_1f_2)(Z \cap X \setminus \{\xbar\}) \subseteq I$, da cui la tesi. \end{enumerate} \end{solution} \begin{exercise} Sia $f: \RR \to \RR$ tale che: \[ f(x) = \system{x + 2x^2 \sin\left(\frac{1}{x}\right) & \se x \neq 0, \\ 0 & \altrimenti.} \] \vskip 0.05in Si mostri che $f$ è continua ovunque e che $D_+ f(0) = 1$. \end{exercise} \begin{solution} Poiché somma di funzioni elementari, $f$ è continua in $(0, \infty)$. Analogamente è continua in $(-\infty, 0)$ dacché è costante in tale intervallo. Affinché allora $f$ sia continua ovunque è sufficiente che si dimostri che è continua anche in $0$. Dal momento che $0$ è un punto di accumulazione sia destro che sinistro di $\RR$, questo equivale a mostrare che il limite destro e sinistro di $f$ esistono in $0$ e coincidono. \\ Si verifica dunque che \[ \lim_{x \to 0^-} f(x) = 0, \qquad \lim_{x \to 0^+} f(x) = \lim_{x \to 0^+} x + \lim_{x \to 0^+} 2x^2 \sin\left(\frac{1}{x}\right) = 0, \] dove si è impiegato il fatto che $\sin\left(\frac{1}{x}\right)$ è limitata in ogni intorno di $0$ e che $2x^2 \tendsto{0^+} 0$; quindi $f$ è continua in $0$, e lo è allora ovunque. \\ Si computa allora la derivata destra di $f$ in $0$: \[ D_+ f(0) = \lim_{h \to 0^+} \frac{h + 2h^2 \sin\left(\frac{1}{h}\right)}{h} = 1 + \lim_{h \to 0^+} 2 h \sin\left(\frac{1}{h}\right) = 1, \] dove si è usato lo stesso argomento di prima per computare $\lim_{h \to 0^+} 2 h \sin\left(\frac{1}{h}\right) = 0$. \end{solution} \begin{theorem} (di de l'Hopital) %TODO: sistemare nome Siano $I$ intervallo e $x_0 \in I$. Sia detto $I' := I \setminus \{ x_0 \}$. Siano $f$, $g : I' \to \RR$ derivabili tali che: \begin{enumerate}[(i)] \item esiste $L := \lim_{x \to x_0} \frac{f'(x)}{g'(x)}$, \item $g' \neq 0$ in $I'$, \item vale che (a) $f(x)$, $g(x) \tendsto{x_0} 0$ oppure che (b) $g(x) \tendsto{x_0} \pm \infty$. \end{enumerate} Allora $\frac{f(x)}{g(x)} \tendsto{x_0} L$. \end{theorem} \begin{proof} Si consideri il caso (a) per $x_0$ finito. Si ponga $f(x_0) = g(x_0) := 0$. Senza perdità di generalità si assuma che $I$ sia un intorno destro di $x_0$. Sia $x \in I \setminus \{x_0\}$, da cui si ricava che $x > x_0$. \\ Si osserva che $\frac{f(x)}{g(x)} = \frac{f(x) - f(x_0)}{g(x)} - g(x_0)$. Per il teorema di Cauchy, esiste allora $\tilde{x} \in (x_0, x)$, in funzione di $x$, tale che $\frac{f(x)}{g(x)} = \frac{f'(\tilde x)}{g'(\tilde x)}$. Allora $\lim_{x \to x_0} \frac{f(x)}{g(x)} = \lim_{x \to x_0} \frac{f'(\tilde x)}{g'(\tilde x)} = \lim_{\tilde x \to x_0} \frac{f'(\tilde x)}{g'(\tilde x)} = L$, dove si è utilizzato che $\tilde x \tendsto{x_0} x_0$ per il teorema del confronto applicato sulla relazione $x_0 < \tilde x < x$. \\ Si consideri ora il caso (b) per $x_0$ finito. Siano $x_1 > x_0$ tali che $x_1 > x > x_0$. Allora vale la seguente identità: \[ \frac{f(x)}{g(x)} = \left( \frac{f(x) - f(x_1)}{g(x) - g(x_1)} + \frac{f(x_1)}{g(x) - g(x_1)} \right) \frac{g(x) - g(x_1)}{g(x)}. \] Si osserva allora che: \[ \lim_{x \to x_0} \frac{f(x)}{g(x)} = \lim_{x \to x_0} \frac{f(x) - f(x_1)}{g(x) - g(x_1)}. \] \end{proof} \begin{remark} È essenziale che $I$ sia un intervallo affinché il teorema di de l'Hopital sia vero. % TODO: scrivi esempio di unione infinita di intervalli (tra x e x^2 + x) \end{remark} \begin{proposition} Sia $I$ un intervallo, sia $x_0 \in I$ e sia $f : I \to \RR$ continua e derivabile dappertutto tranne che in $x_0$. Se esiste $L := \lim_{x \to x_0} f'(x)$, allora $f'(x_0) = L$. \end{proposition} \begin{proof} Si consideri il rapporto incrementale $\frac{f(x) - f(x_0)}{x - x_0}$. Allora, per $x \to x_0$, per il teorema di de l'Hopital, $f'(x_0) = \lim_{x \to x_0} f'(x)$. \end{proof} \begin{theorem} (sullo sviluppo di Taylor) Sia $I$ un intervallo e sia $\xbar \in I$. Sia $f : I \to \RR$ e sia $d \in \NN$. Sia $f$ derivabile $d-1$ dappertutto e sia derivabile $d$ volte in $\xbar$. Allora, detti \[ P_d(h) = f(\xbar) + f'(\xbar) h + \ldots + \frac{f^{(d)}(\xbar)}{d!} h^d, \] \[ R_d(h) = f(\xbar + h) - P_d(h), \] \begin{enumerate}[(a)] \item $R_d(h) = o(h^d)$ per $h \to 0$, \item se $f$ è derivabile $d$ volte su $I$ e $d+1$ volte in $\xbar$, allora $R_d(h) = O(h^{d+1})$ per $h \to 0$ e $\frac{R_d(h)}{h^{d+1}} \to \frac{f^{(d+1)}(\xbar)}{(d+1)!}$, \item se $f$ è derivabile $d+1$ volte su $I$, allora $\forall h \mid \xbar + h \in I$, $\exists \tilde x \in [\xbar, \xbar + h] \mid R_d(h) = \frac{f^{(d+1)}(\tilde x)}{(d+1)!}$ (\textit{formula del resto di Lagrange}), \item se $f \in C^{d+1}$, allora $R_d(h) = \frac{1}{d!} \int_0^h (h-t)^d f^{(d+1)}(\xbar + t) \, dt$ (\textit{formula integrale}). \end{enumerate} \end{theorem} \begin{proof} % TODO: dimostrazione farlocca, migliorarla (d) Si assuma $\xbar = 0$ e $f \in C^{d+1}$. Innanzitutto si osserva che la tesi è equivalente a mostrare che $f(h) = P_d(h) + \frac{1}{d!} \int_0^h (h-t)^d f^{(d+1)}(t) \, dt$. \\ Se $d=0$, $f(h) = f(0) + \int_0^h f'(t) \,dt$ (teorema fondamentale del calcolo integrale). $f(h) = f(0) + \abs{-(h-t) f'(h)}_0^h + \int_0^h (h-t) f''(t) \, dt$. [...] % TODO: si continua per induzione \end{proof} \begin{proposition} Sia $f : I \to \RR$ derivabile. Sia $\xbar \in I$ tale che $f'(\xbar) = 0$ ed esista $f''(\xbar)$. Allora: \begin{enumerate}[(i)] \item $f''(\xbar) > 0 \implies \xbar$ è un punto di minimo locale stretto, \item $f''(\xbar) < 0 \implies \xbar$ è un punto di massimo locale.stretto, \item $\xbar$ è un punto di minimo locale $\implies f''(\xbar) \geq 0$, \item $\xbar$ è un punto di massimo locale $\implies f''(\xbar) \leq 0$. \end{enumerate} \end{proposition} \begin{proof} Per lo sviluppo di Taylor, $f(\xbar + h) = f(\xbar) + f'(\xbar) h + \frac{1}{2} f''(\xbar) h^2 + o(h^2) \implies \frac{f(\xbar + h) - f(\xbar)}{h^2} = \frac{1}{2} f''(\xbar) + o(1)$ (infinitesimo). Allora $\frac{f(\xbar + h) - f(\xbar)}{h^2} \tendsto{h} L > 0$. Quindi permanenza del segno. \end{proof} \end{document}